Foros-FIUBA Foros HostingPortal
 FAQ  •  Buscar  •  Wiki  •  Apuntes  •  Planet  •  Mapa  •  Eyeon  •  Chat
Preferencias  •  Grupos de Usuarios
Registrarse  •  Perfil  •  Entrá para ver tus mensajes privados  •  Login
Ver tema siguiente
Ver tema anterior

Responder al tema Ver tema anteriorEnviar por mail a un amigo.Mostrar una Lista de los Usuarios que vieron este TemaGuardar este Tema como un archivoPrintable versionEntrá para ver tus mensajes privadosVer tema siguiente
Autor Mensaje
koreano
Nivel 9



Registrado: 15 Jul 2010
Mensajes: 1796

Carrera: No especificada
blank.gif
MensajePublicado: Jue Oct 27, 2011 2:41 pm  Asunto: Nota Guía 6: Magnetismo estacionario en medios materiales Responder citandoFin de la PáginaVolver arriba

Guía 6: Magnetismo estacionario en medios materiales

Enunciados: http://materias.fi.uba.ar/6203/Download/Problemas%20y%20Laboratorio/guia%206-2C%202009.pdf

Para resolver los siguientes ejercicios tomamos en cuenta la aproximación de que todo el flujo queda confinado dentro del circuito magnético. Esto se debe a que la permeabilidad de los núcleos es muchísimo mas alta que la del vacío, por lo tanto la cantidad de líneas de B que existen fuera del material es despreciable. Por otro lado, en el caso de los entrehierros, donde no hay material, sino vacío, hacemos la suposicion de que todo el flujo vuelve a ingresar al circuito una vez que atraviesa el entrehierro (ver ej. 3).

1) a) Teniendo en cuenta las hipótesis listadas al principio, para resolver el ejercicio elegimos una curva "amperiana" C, que vendría a ser el equivalente a una superficie Gaussiana cuando trabajabamos con electricidad y la Ley de Gauss. La curva se elige de manera que sea paralela al campo H (y por lo tanto B, ya que H y B son paralelos) en todo momento. De esta manera, podemos reemplazar la integral con un simple producto.

Image

Notar que de antemano sabemos el sentido que va a tener el campo B generado por el bobinado, dado por la regla de la mano derecha, y elegimos la curva en el mismo sentido. Planteamos ahora la ley de Ampere:

[tex]\oint_C \vec{H}\cdot\vec{dl} = NI[/tex]

[tex]HL = NI[/tex]

Usamos la relación constitutiva [tex]\vec{B} = \mu \vec{H}[/tex], donde [tex]\mu = \mu_0 \mu_r[/tex].

[tex]\frac{B}{\mu} L = NI[/tex]

Sabiendo que [tex]d\Phi_B = \vec{B} \cdot \vec{dA}[/tex], como asumimos que el campo es normal a la sección en todo momento, reemplazamos [tex]B = \frac{\Phi_B}{A}[/tex]. Resulta:

[tex]\frac{\Phi_B}{A \mu} L = NI[/tex]

O lo que es lo mismo:

[tex]\Phi_B \frac{L}{A \mu} = NI[/tex]

De acá podemos despejar el flujo. En este momento introducimos una notación nueva (que solo lo podemos hacer con la debida justificación utilizando la ley de Ampere y las hipótesis mencionadas al principio).

Reluctancia:

* [tex]R = \frac{L}{A \cdot \mu}[/tex] (A = Area de sección, L = longitud, [tex]\mu[/tex] = permeabilidad)

Fuerza magnetomotriz:

*[tex]F_m = N \cdot I[/tex] (N = numero de vueltas, I = corriente)


Con estas nuevas definiciones, la ecuación anterior se puede escribir como:

[tex]\Phi_B R = F_m[/tex]

Despejando:

[tex]I = \frac{ \Phi_B \cdot L }{N \mu_r \mu_0 A}[/tex]

[tex]I = \frac{ 10^{-5} Wb \cdot 0.4 m }{200 \cdot 1000 \cdot 4\pi10^{-7} N/A^2 \cdot 10^{-4} m^2} = 0.159\,A[/tex]

b) Conocido el flujo de B y el área de sección podemos averiguar la magnitud de B:

[tex]B = \frac{\Phi_B}{A} = 0.1 \, T[/tex]

Para el cálculo de H:

[tex]B = \mu H[/tex]
[tex]H = \frac{B}{\mu_r \mu_0} = 79.6 \, A/m[/tex]

También: [tex]\oint \vec{H} \cdot \vec{dl} = NI[/tex]

[tex]H = \frac{NI}{L} = 79.6 \, A/m[/tex]

Resta calcular M:

[tex]B = \mu_0(H + M)[/tex]
[tex]M = \frac{B}{\mu_0} - H = 79500 \, A/m[/tex]

Notar la enorme contribución del M, producto del material ferromagnético de alta permeabilidad relativa.

c) El flujo concatenado por el arrollamiento es [tex]N \cdot \Phi_B = 0.002 \, Wb[/tex]

d) El coeficiente de autoinducción lo podemos calcular de varias maneras. Como no tenemos datos precisos sobre la geometría del corte de sección, pero tenemos el flujo concatenado por la espira, podemos calcular la autoinducción como:

[tex]L = \frac{\Phi_{conc}}{I} = \frac{0.002 \, Wb}{0.159 \, A} = 0.0126 \, H[/tex]

La energía del campo magnético es idéntica la cantidad de energía necesiaria para mantenerlo mediante la corriente: [tex]E_B = \frac{1}{2} L I^2 = 159.3 \, \mu J[/tex]



2) a) Idéntico al anterior pero ahora el dato es la fuerza magnetomotriz.

[tex]\Phi_B R = F_m[/tex]

[tex]\Phi_B = NI \frac{A \mu_r \mu_0}{L} = 500 \cdot 2\,A \frac{10^{-4} m^2 \cdot 1000 \cdot 4\pi10^{-7} N/A^2}{0.4 m} = 3.14 \cdot 10^{-4} \, Wb[/tex]

b) [tex]B = \frac{\Phi_B}{A} = 3.14 \, T[/tex]

[tex]H = \frac{B}{\mu_r \mu_0} = 2499 \, A/m[/tex]

[tex]M = \frac{B}{\mu_0} - H = 2.496\cdot10^6 \, A/m[/tex]



3) El entrehierro de 1mm actúa como una "resistencia" adicional en el circuito. Mas específicamente, como una reluctancia mas. Sin embargo, es útil pensarlo como una resistencia ya que el aire tiene menos permeabilidad que el material ferromagnético. De hecho, se considera que el aire tiene aproximadamente [tex]\mu_r = 1[/tex]. Por lo tanto, las líneas de B tienen mas dificultad en fluir por el aire y para la misma fuerza magnetomotriz se va a obtener menos flujo en el circuito. Es el análogo a las resistencias en un circuito, siendo el flujo magnético el equivalente a la corriente. Hacemos la suposición de que si bien hay efectos de frontera en las interfazes, todo el flujo sigue quedando confinado en el circuito ya que la separacion es muy pequeña.

Image

Planteamos las ecuaciones, pero ahora:

[tex]L = L_n + L_e[/tex] (n = núcleo, e = entrehierro)
[tex]0.4\,m = L_n + 0.001\,m[/tex]

[tex]L_n = 0.399\,m[/tex]
[tex]L_e = 0.001\,m[/tex]

Asociado a cada sección hay una permeabilidad:
[tex]\mu_n = 1000\cdot\mu_0[/tex]
[tex]\mu_e = \mu_0[/tex]

Y el área de sección es constante a lo largo del circuito. Tomamos nuevamente una curva C en el sentido de la corriente y planteamos la Ley de Ampere:

Image

[tex]\oint_C \vec{H}\cdot\vec{dl} = NI[/tex]

Sin embargo, ahora debemos partir la integral en dos para cada material.

[tex]H_n L_n + H_e L_e = NI[/tex]

Una vez mas usamos la relación constitutiva [tex]\vec{B} = \mu \vec{H}[/tex], donde [tex]\mu = \mu_0 \mu_r[/tex].

[tex]\frac{B_n}{\mu_n} L_n + \frac{B_e}{\mu_e} L_e = NI[/tex]

Reemplazamos ahora la relación [tex]B = \frac{\Phi_B}{A}[/tex]. Resulta:

[tex]\Phi_{B_n} \frac{L_n}{A \mu_n} + \Phi_{B_e} \frac{L_e}{A \mu_e}= NI[/tex]

Sin embargo, de acuerdo a la Ley de Gauss para el magnetismo, el flujo neto de B a través de cualquier superficie cerrada es 0. Por lo tanto, el flujo de B debe ser el mismo a través de cualquier sección del circuito magnético. En otras, palabras, el flujo se conserva y [tex]\Phi_{B_n} = \Phi_{B_e} = \Phi_B[/tex]. Reemplazando, podemos sacar factor común:

[tex]\Phi_B \left(\frac{L_n}{A \mu_n} + \frac{L_e}{A \mu_e}\right)= NI[/tex]

Notar que las reluctancias del núcleo y del material se suman. Es análogo a tener dos resistencias en serie en los circuitos eléctricos de CC. Podríamos haber planteado directamente lo que se conoce como la ley de Kirchoff para circuitos magnéticos:

[tex]\Phi_B \left(\sum_i R_i \right)= \sum F_i[/tex]

Donde, por analogía con CC, vemos que las reluctancias forman el papel de las resistencias, el flujo es la nueva "corriente" y la fuerza magnetomotriz toma el papel del voltaje. Recordamos la definición de los nuevos términos:

Reluctancia i-ésima:

* [tex]R_i = \frac{L_i}{A_i \cdot \mu_i}[/tex] (A = Area de sección, L = longitud, [tex]\mu[/tex] = permeabilidad)

Fuerza magnetomotriz i-esima:

*[tex]F_i = N_i \cdot I_i[/tex] (N = numero de vueltas, I = corriente)

Notar que en el caso de la FMM, debemos tener en cuenta el signo (especialmente cuando hay mas de un bobinado). Si tenemos por ejemplo dos bobinados enrollados en sentido opuesto, deben tener signo opuesto cuando los sumamos. Es análogo a elegir arbitrariamente un sentido para la corriente y tomar las pilas orientadas en un sentido como subidas de potencial y las orientadas en sentido opuesto como caídas de potencial.

Volviendo a nuestro caso, esquemáticamente podemos visualizar la analogía:

Image

[tex]\sum R = \frac{L_n}{A \cdot \mu_n} + \frac{L_e}{A \cdot \mu_e}[/tex]

[tex]\sum F = NI[/tex]

De aquí se resuelvo exactamente igual que el ejercicio [tex]1)[/tex]. Notar que, de alguna forma, en la ecuación de Kirchoff ya se toma a [tex]\Phi_B[/tex] como factor común. Esta es la expresión algebraica que indica que el flujo queda siempre confinado al circuito y es único para todos los materiales.

Cuando hacemos los ejercicios en el exámen, es altamente improbable que nos lo den por correcto si no explicamos el concepto de reluctancia y lo usamos "ciegamente". Por lo tanto, es importante explicar la aplicación de la Ley de Ampere, así como las hipótesis realizadas para resolver el ejercicio. De hecho, el concepto de reluctancia o fuerza magnetomotriz aquí presentados pueden ser ignorados para resolver el ejercicio, haciendo uso únicamente de las leyes de Maxwell y las condiciones de frontera (ademas de las hipótesis sobre el circuito).


4) Sabemos de la guía anterior que el campo del toroide es variable con la distancia al centro del toroide:

[tex]B(r) = \frac{\mu NI}{2 \pi r}[/tex]

Tenemos una sección cuadrada por lo que para evaluar el flujo de cada espira vamos a tener que integrar. Tomando un sistema de coordenadas cilíndricas en el que el toroide reposa sobre el plano r-tita:

[tex]R = R_2 - R_1 = 0.01 \, m[/tex]

[tex]\Phi_B = \int_0^R \int_{R_1}^{R_2} \frac{\mu NI}{2 \pi r} \, dr \, dz[/tex]

[tex]\Phi_B = \frac{\mu NI}{2 \pi} R \cdot ln(\frac{R_2}{R_1}) = 1.9\cdot 10^{-4} \, Wb[/tex]

El flujo concatenado por el bobinado es [tex]N\Phi_B[/tex].

b) Se ve rápidamente que el campo varía como [tex]\frac{1}{r}[/tex] por lo que los valores máximos van a estar sobre el radio interior del toroide.

[tex]B_{max} = B(R_1)[/tex]
[tex]H_{max} = \mu B_{max}[/tex]
[tex]M_{max} = \frac{B_{max}}{\mu_0} - H_{max}[/tex]

Image

c) El flujo si se utiliza un núcleo de aire (alias, sin núcleo en especial) va a ser mucho menor ya que [tex]\mu = \mu_0[/tex]. Como es lineal en la permeabilidad, sería 800 veces menor.

d) Se procede igual que en el ejercicio 3)


5) Se resuelve igual que el ejercicio 1) y 3). Dejo planteada la situación:

[tex]L_1 = 0.4\,m[/tex]
[tex]L_2 = 0.1\,m[/tex]

[tex]A_1 = 1 \cdot 10^{-4} \, m^2[/tex]
[tex]A_2 = 1.5 \cdot 10^{-4} \, m^2[/tex]

Asociado a cada sección hay una permeabilidad:
[tex]\mu_1 = 1000\cdot\mu_0[/tex]
[tex]\mu_2 = 2000\cdot\mu_0[/tex]

Planteamos:

[tex]\Phi_B \sum R = F_m = NI[/tex]

[tex]\sum R = \frac{L_1}{A_1 \cdot \mu_1} + \frac{L_2}{A_2 \cdot \mu_2}[/tex]

El resto sale con fritas.


6) Siguiendo con la analogía de los círcuitos magnéticos y los circuitos de CC, los dos núcleos serían como dos reluctancias en paralelo. Esto quiere decir que cada una establece un flujo propio independiente del otro (excepto que en ningún momento se juntan en un nodo en este caso) y que ambas están sujetas a la misma FMM, así como dos resistencias en paralelo estarían sujetas a la misma diferencia de potencial.

Por lo tanto si queremos resolver el problema desde 0, tenemos que plantear dos curvas amperianas, una para cada material y resolver independientemente. La otra manera es considerar directamente las reluctancias por separado. Esquemáticamente tendriamos el siguiente "circuito":

Image

De donde se deducen las siguientes ecuaciones:

[tex]\Phi_1 R_1 = NI[/tex]

[tex]\Phi_2 R_2 = NI[/tex]

Y resolver como ya sabemos (ver ej 1 y 3). De las dos maneras debemos llegar al mismo resultado.



7) Tenemos el dato del vector magnetización, que existe únicamente dentro del material. Por otro lado, para tener un imán permanente (es decir, sin corrientes de conducción que lo "soporten"), debemos estar en el segundo (o cuarto, dependiendo de la dirección) cuadrante de la curva de histéresis. Esto se deduce del hecho de que, la no haber corrientes de conducción, la circulación de H es 0. Por lo tanto, si bien en el exterior se cumple la relación constitutiva entre H y B (M es 0 y [tex]\mu_r = 1[/tex]), en el interior del imán, H debe ser negativo ya que [tex]\oint \vec{H} \cdot \vec{dl} = 0[/tex] y H es positivo en el exterior del imán. Resulta:

Image



8) 9) 10) Estos ejercicios se resuelven igual que los primeros excepto que la relación entre B y H no es lineal sino que viene dada por tablas. Por ejemplo, en el 8), si B debe ser 1T vemos que el gráfico indica que H tiene que ser 79.6 A/m. Entonces utilizamos dicha relación para plantear nuestras ecuaciones. Así sucesivamente con los demás ejercicios en los que tenemos que hacer uso de las tablas de datos que se dan al final de la guía.


--------------

Comentarios son bienvenidos siempre, correciones o truquitos.




Última edición por koreano el Dom Jun 24, 2012 7:48 pm, editado 5 veces
   OcultoGalería Personal de koreanoVer perfil de usuarioEnviar mensaje privado
Jackson666
Nivel 9


Edad: 37
Registrado: 01 Feb 2009
Mensajes: 1980
Ubicación: Martínez
Carrera: Electricista
CARRERA.electrica.3.jpg
MensajePublicado: Jue Oct 27, 2011 5:09 pm  Asunto:  (Sin Asunto) Responder citandoFin de la PáginaVolver arriba

Muy piola wachín. Gracias nuevamente Very Happy.


Aries Género:Masculino Gato OfflineGalería Personal de Jackson666Ver perfil de usuarioEnviar mensaje privado
La gallina Pipa
Nivel 8


Edad: 84
Registrado: 16 Jul 2010
Mensajes: 611
Ubicación: Calle Falsa 123
Carrera: No especificada
argentina.gif
MensajePublicado: Jue Oct 27, 2011 10:26 pm  Asunto:  (Sin Asunto) Responder citandoFin de la PáginaVolver arriba

che, me parece a mi o la reluctancia la paso de largo Santiago?

_________________
Nosotros... Chat FIUBA!

Piscis Género:Masculino Dragón OcultoGalería Personal de La gallina PipaVer perfil de usuarioEnviar mensaje privadoMSN Messenger
koreano
Nivel 9



Registrado: 15 Jul 2010
Mensajes: 1796

Carrera: No especificada
blank.gif
MensajePublicado: Vie Oct 28, 2011 8:16 pm  Asunto:  (Sin Asunto) Responder citandoFin de la PáginaVolver arriba

Completé el problema 7) Smile


   OcultoGalería Personal de koreanoVer perfil de usuarioEnviar mensaje privado
mascheraco
Nivel 3



Registrado: 03 Jul 2011
Mensajes: 49

Carrera: Electrónica
argentina.gif
MensajePublicado: Dom Nov 20, 2011 12:35 pm  Asunto:  (Sin Asunto) Responder citandoFin de la PáginaVolver arriba

Segun nos dijo Donofrio, quedaron en que no iban a explicar reluctancia, o por lo menos que no lo iban a tomar. Asi que quedate tranquilo jaja


 Género:Masculino  OfflineGalería Personal de mascheracoVer perfil de usuarioEnviar mensaje privado
JinnKaY
Nivel 9


Edad: 32
Registrado: 16 Jul 2010
Mensajes: 1445

Carrera: Electrónica y Mecánica
CARRERA.electronica.5.gif
MensajePublicado: Dom Nov 20, 2011 6:03 pm  Asunto:  (Sin Asunto) Responder citandoFin de la PáginaVolver arriba

Puede ser que en varios puntos tengas mal el resultado por un factor de 10?

[tex]M = \frac{B}{\mu_0} - H = 2496\cdot10^4 \, A/m[/tex]

[tex]M = \frac{3,14 \frac{wb}{m^2}}{4\pi 10^{-7}\frac{wb}{m^2}} - 2499 \frac{A}{m}[/tex]

[tex]M = 2498732,607 \frac{A}{m}- 2499 \frac{A}{m}=2496233,607\frac{A}{m} = 2496,23*10^3[/tex]

Y te dio a la cuarta ^^ creo que paso algo parecido en otros puntos donde calculas el M (en uno me dio 79,57 y a vos 7950)

_________________
Imagehttp://tinyurl.com/8y3ghjgImage

Image


[tex][|0|.................|25|.................|50|.................|75|.................|100|][/tex]
[tex][|||||||||||||||||||||||||||||||||||..............................................................][/tex]

Virgo Género:Masculino Cabra OfflineGalería Personal de JinnKaYVer perfil de usuarioEnviar mensaje privado
koreano
Nivel 9



Registrado: 15 Jul 2010
Mensajes: 1796

Carrera: No especificada
blank.gif
MensajePublicado: Dom Nov 20, 2011 6:19 pm  Asunto:  (Sin Asunto) Responder citandoFin de la PáginaVolver arriba

Ahí los revisé, no sé por qué habían quedado mal O_o


   OcultoGalería Personal de koreanoVer perfil de usuarioEnviar mensaje privado
JinnKaY
Nivel 9


Edad: 32
Registrado: 16 Jul 2010
Mensajes: 1445

Carrera: Electrónica y Mecánica
CARRERA.electronica.5.gif
MensajePublicado: Dom Nov 20, 2011 8:02 pm  Asunto:  (Sin Asunto) Responder citandoFin de la PáginaVolver arriba

koreano escribió:
Ahí los revisé, no sé por qué habían quedado mal O_o


Creo que el error viene por el lado de plantearlo con B y H ... yo prefiero usar :

[tex]\vec{M}=\vec{H}(\mu_r -1)[/tex] que sale de reemplazar la relacion entre B y H en la expresion que usas .... son muchas menos cuentas de esta manera ^^ y la probabilidad de cometer un error de cuentas es menor.

_________________
Imagehttp://tinyurl.com/8y3ghjgImage

Image


[tex][|0|.................|25|.................|50|.................|75|.................|100|][/tex]
[tex][|||||||||||||||||||||||||||||||||||..............................................................][/tex]

Virgo Género:Masculino Cabra OfflineGalería Personal de JinnKaYVer perfil de usuarioEnviar mensaje privado
Trigger
Nivel 8



Registrado: 06 Ago 2008
Mensajes: 524

Carrera: Industrial
argentina.gif
MensajePublicado: Sab Dic 10, 2011 11:00 am  Asunto:  (Sin Asunto) Responder citandoFin de la PáginaVolver arriba

En el ejercicio 6, que planteo en la ley de Ampere? Yo se que por condiciones de borde, los componentes tangenciales de H son iguales y los componentes normales de B tambien. Pero como plasmo eso en Ampere? lo pongo vectorialmente?


 Género:Masculino  OfflineGalería Personal de TriggerVer perfil de usuarioEnviar mensaje privado
Jackson666
Nivel 9


Edad: 37
Registrado: 01 Feb 2009
Mensajes: 1980
Ubicación: Martínez
Carrera: Electricista
CARRERA.electrica.3.jpg
MensajePublicado: Sab Dic 10, 2011 11:54 am  Asunto:  (Sin Asunto) Responder citandoFin de la PáginaVolver arriba

Suponiendo que las líneas de H copian aproximadamente la forma del núcleo y sabiendo que las componentes tangenciales de H son iguales a un lado y a otro de la interfaz, tenes que H1 = H2 = H, por tener H sólo componente tangencial a la interfaz.

Entonces, tenes un único H y no dos. Planteas la ley de Ampère, [tex]\oint_{\mathcal{C}}{\vec{H} \cdot d\vec{l}} = I_{\text{conc}} = N \cdot I[/tex]. Tenes la longitud media, sacas la magnitud de H. Tenes las permeabilidades magnéticas relativas, que son un número por ser el material lineal, entonces tenes los campos [tex]\vec{B}_{1}, \; \vec{B}_{2}[/tex].

Tenes la sección de ambos "cachos" de material y ambos campos de inducción. Multiplicas el módulo del campo por la sección correspondiente, tenes el flujo a cada lado. Lo sumas, tenes el flujo total en 1 sección. Lo multiplicas por N, tenes el flujo concatenado.


Aries Género:Masculino Gato OfflineGalería Personal de Jackson666Ver perfil de usuarioEnviar mensaje privado
Trigger
Nivel 8



Registrado: 06 Ago 2008
Mensajes: 524

Carrera: Industrial
argentina.gif
MensajePublicado: Dom Dic 11, 2011 9:09 pm  Asunto:  (Sin Asunto) Responder citandoFin de la PáginaVolver arriba

Gracias Jackson!!


 Género:Masculino  OfflineGalería Personal de TriggerVer perfil de usuarioEnviar mensaje privado
SofiaC
Nivel 4


Edad: 32
Registrado: 10 Feb 2010
Mensajes: 60

Carrera: Química
argentina.gif
MensajePublicado: Vie Dic 16, 2011 11:22 pm  Asunto:  (Sin Asunto) Responder citandoFin de la PáginaVolver arriba

Tengo una duda de los ejercicios 5 y 6. En el caso del 5, no puedo decir que los campos B son iguales? Por condición de frontera, como las componentes son normales en B, entonces B1 = B2... En el 6, lo mismo pero con H, con las componentes tangenciales. Se puede hacer eso o estoy errando feo?

_________________
Image

So if you have a minute, why don´t we go talk about it somewhere only we know?

Image

Tauro Género:Femenino Cabra OfflineGalería Personal de SofiaCVer perfil de usuarioEnviar mensaje privado
koreano
Nivel 9



Registrado: 15 Jul 2010
Mensajes: 1796

Carrera: No especificada
blank.gif
MensajePublicado: Sab Dic 17, 2011 8:31 am  Asunto:  (Sin Asunto) Responder citandoFin de la PáginaVolver arriba

Si partís de la base de que el flujo es constante, entonces B no puede ser constante, porque [tex]\Phi_B = B \cdot A[/tex]. Cuando tenés un cambio de sección, [tex]A[/tex] cambia. Si [tex]\Phi_B[/tex] tiene que ser el mismo, entonces [tex]B[/tex] se tiene que ajustar para que el producto sea invariante.

Lo que decís de las componentes normales de B es correcto, pero te estás olvidando que en la interfaz entre dos secciones de distinta área entran en efecto las componentes tangenciales y la magnitud de B tiene en cuenta ambas contribuciones (pitágoras). Acá tenés un esquemita, no sé si se entiende:

Image

Si [tex]\Phi_B = k[/tex] entonces [tex]B_1 A_1 = B_2 A_2[/tex] pero [tex]A_1 \neq A_2[/tex] entonces [tex]B_1 \neq B_2[/tex]. A la derecha es un 'zoom' del cambio de sección donde podés imaginarte como contribuirían a B las componentes tangenciales.

No se me ocurre otra manera de explicarlo. Creo que la manera mas fácil de pensarlo en este caso es pensar a [tex]B[/tex] como "densidad de flujo magnético" y no como "campo magnético".


   OcultoGalería Personal de koreanoVer perfil de usuarioEnviar mensaje privado
SofiaC
Nivel 4


Edad: 32
Registrado: 10 Feb 2010
Mensajes: 60

Carrera: Química
argentina.gif
MensajePublicado: Sab Dic 17, 2011 4:38 pm  Asunto:  (Sin Asunto) Responder citandoFin de la PáginaVolver arriba

Gracias!!!! Me diste una re mano!!

_________________
Image

So if you have a minute, why don´t we go talk about it somewhere only we know?

Image

Tauro Género:Femenino Cabra OfflineGalería Personal de SofiaCVer perfil de usuarioEnviar mensaje privado
agusvazquez
Nivel 6


Edad: 33
Registrado: 10 Mar 2010
Mensajes: 252

Carrera: Informática y Sistemas
argentina.gif
MensajePublicado: Vie Feb 03, 2012 5:41 pm  Asunto:  (Sin Asunto) Responder citandoFin de la PáginaVolver arriba

Alta guia!!! Deberian ponerlaa como sticky (si es que no lo esta aun, la verdad no me fije), muy buena!!!!!!

Gracias che me salvaste la vida

_________________
Mis Aplicaciones para Android
Image
Material para la facu:
www.agustinvazquez.net/facultad

Geminis  Caballo OfflineGalería Personal de agusvazquezVer perfil de usuarioEnviar mensaje privadoEnviar emailMSN Messenger
Mostrar mensajes de anteriores:      
Responder al tema Ver tema anteriorEnviar por mail a un amigo.Mostrar una Lista de los Usuarios que vieron este TemaGuardar este Tema como un archivoPrintable versionEntrá para ver tus mensajes privadosVer tema siguiente

Ver tema siguiente
Ver tema anterior
Podés publicar nuevos temas en este foro
No podés responder a temas en este foro
No podés editar tus mensajes en este foro
No podés borrar tus mensajes en este foro
No podés votar en encuestas en este foro
No Podéspostear archivos en este foro
No Podés bajar archivos de este foro


Todas las horas son ART, ARST (GMT - 3, GMT - 2 Horas)
Protected by CBACK CrackerTracker
365 Attacks blocked.

Powered by phpBB2 Plus, phpBB Styles and Kostenloses Forum based on phpBB © 2001/6 phpBB Group :: FI Theme :: Mods y Créditos

Foros-FIUBA está hosteado en Neolo.com Cloud Hosting

[ Tiempo: 0.2846s ][ Pedidos: 20 (0.1607s) ]